Confidence interval of a biased estimator that's asymptotiacally unbiased.












0












$begingroup$


Suppose I have an estimator $hat{theta}_n$ of $theta$ where $mathbb{E}[hat{theta}_n] neq theta$, but I do know that $displaystylelim_{n to infty} mathbb{E}[hat{theta}_n] = theta$.



Also suppose that $hat{theta}_n sim mathcal{N}(mu_n, sigma_n^2)$ and from the above we then have that $displaystylelim_{ntoinfty} mu_n = theta$.



Is there any way to construct confidence intervals of the form with probability $geq 1 - delta$, $theta in [text{lower bound}, text{upper bound}]$?



I was trying to find an upper bound on
$$
mathbb{P}(|hat{theta}_n - theta| > varepsilon) < text{???}
$$

which we could then invert to get something of the form "with probability $geq 1 - delta$, $theta in [L(hat{theta}_n, n), U(hat{theta}_n, n)]$ where $L$ and $U$ are the lower and upper bounds.



Since for finite $n$, $hat{theta}_n$ isn't unbiased, we can't use standard Gaussian tail bounds or anything so I'm not sure what the best approach is here.










share|cite|improve this question









$endgroup$

















    0












    $begingroup$


    Suppose I have an estimator $hat{theta}_n$ of $theta$ where $mathbb{E}[hat{theta}_n] neq theta$, but I do know that $displaystylelim_{n to infty} mathbb{E}[hat{theta}_n] = theta$.



    Also suppose that $hat{theta}_n sim mathcal{N}(mu_n, sigma_n^2)$ and from the above we then have that $displaystylelim_{ntoinfty} mu_n = theta$.



    Is there any way to construct confidence intervals of the form with probability $geq 1 - delta$, $theta in [text{lower bound}, text{upper bound}]$?



    I was trying to find an upper bound on
    $$
    mathbb{P}(|hat{theta}_n - theta| > varepsilon) < text{???}
    $$

    which we could then invert to get something of the form "with probability $geq 1 - delta$, $theta in [L(hat{theta}_n, n), U(hat{theta}_n, n)]$ where $L$ and $U$ are the lower and upper bounds.



    Since for finite $n$, $hat{theta}_n$ isn't unbiased, we can't use standard Gaussian tail bounds or anything so I'm not sure what the best approach is here.










    share|cite|improve this question









    $endgroup$















      0












      0








      0





      $begingroup$


      Suppose I have an estimator $hat{theta}_n$ of $theta$ where $mathbb{E}[hat{theta}_n] neq theta$, but I do know that $displaystylelim_{n to infty} mathbb{E}[hat{theta}_n] = theta$.



      Also suppose that $hat{theta}_n sim mathcal{N}(mu_n, sigma_n^2)$ and from the above we then have that $displaystylelim_{ntoinfty} mu_n = theta$.



      Is there any way to construct confidence intervals of the form with probability $geq 1 - delta$, $theta in [text{lower bound}, text{upper bound}]$?



      I was trying to find an upper bound on
      $$
      mathbb{P}(|hat{theta}_n - theta| > varepsilon) < text{???}
      $$

      which we could then invert to get something of the form "with probability $geq 1 - delta$, $theta in [L(hat{theta}_n, n), U(hat{theta}_n, n)]$ where $L$ and $U$ are the lower and upper bounds.



      Since for finite $n$, $hat{theta}_n$ isn't unbiased, we can't use standard Gaussian tail bounds or anything so I'm not sure what the best approach is here.










      share|cite|improve this question









      $endgroup$




      Suppose I have an estimator $hat{theta}_n$ of $theta$ where $mathbb{E}[hat{theta}_n] neq theta$, but I do know that $displaystylelim_{n to infty} mathbb{E}[hat{theta}_n] = theta$.



      Also suppose that $hat{theta}_n sim mathcal{N}(mu_n, sigma_n^2)$ and from the above we then have that $displaystylelim_{ntoinfty} mu_n = theta$.



      Is there any way to construct confidence intervals of the form with probability $geq 1 - delta$, $theta in [text{lower bound}, text{upper bound}]$?



      I was trying to find an upper bound on
      $$
      mathbb{P}(|hat{theta}_n - theta| > varepsilon) < text{???}
      $$

      which we could then invert to get something of the form "with probability $geq 1 - delta$, $theta in [L(hat{theta}_n, n), U(hat{theta}_n, n)]$ where $L$ and $U$ are the lower and upper bounds.



      Since for finite $n$, $hat{theta}_n$ isn't unbiased, we can't use standard Gaussian tail bounds or anything so I'm not sure what the best approach is here.







      probability statistics probability-distributions concentration-of-measure






      share|cite|improve this question













      share|cite|improve this question











      share|cite|improve this question




      share|cite|improve this question










      asked Nov 30 '18 at 4:29







      user610084





























          1 Answer
          1






          active

          oldest

          votes


















          0












          $begingroup$

          At the moment your stated distribution for $hat{theta}_n$ does not specify the relationship between the moments $mu_n$ and $sigma_n^2$ and the true parameter $theta$. This means that there is presently insufficient information to form a pivotal quantity that would be the basis for a confidence interval. If we make some assumptions about the relationship of these moments to the underlying parameter then we can form a pivotal quantity and derive an appropriate confidence interval formula.



          For simplicity, I will assume that the variance $sigma_n^2$ is unrelated to the underlying parameter, and that the mean is related to the underlying parameter through a monotonic function. That is, you have $mu_n = f_n(theta)$ via some monotonically increasing function $f_n$ (we will denote the inverse function by $g_n = f_n^{-1}$). Then you can write the pivotal quantity:



          $$frac{hat{theta}_n - f_n(theta)}{sigma_n} sim mathcal{N}(0,1).$$



          You have:



          $$begin{equation} begin{aligned}
          1-alpha
          &= mathbb{P} Big( z_{alpha/2} leqslant frac{hat{theta}_n - f_n(theta)}{sigma_n} leqslant z_{alpha/2} Big) \[6pt]
          &= mathbb{P} Big( sigma_n cdot z_{alpha/2} leqslant hat{theta}_n - f_n(theta) leqslant sigma_n cdot z_{alpha/2} Big) \[6pt]
          &= mathbb{P} Big( hat{theta}_n - sigma_n cdot z_{alpha/2} leqslant f_n(theta) leqslant hat{theta}_n + sigma_n cdot z_{alpha/2} Big) \[6pt]
          &= mathbb{P} Big( g_n(hat{theta}_n - sigma_n cdot z_{alpha/2}) leqslant theta leqslant g_n(hat{theta}_n + sigma_n cdot z_{alpha/2}) Big). \[6pt]
          end{aligned} end{equation}$$



          In the above equations we have taken $hat{theta}_n$ to be a random variable (i.e., prior to fixing the data). Once the data is observed this statistic becomes a fixed value $hat{theta}_{n,text{obs}}$ and we have the $1-alpha$ level confidence interval:



          $$text{CI}_theta(1-alpha) = Big[ g_n(hat{theta}_{n,text{obs}} - sigma_n cdot z_{alpha/2}), text{ } g_n(hat{theta}_{n,text{obs}} + sigma_n cdot z_{alpha/2}) Big].$$



          (Note that this formula is for a monotonically increasing function $f_n$. If the function were instead monotonically decreasing then the bounds of the confidence interval would be reversed.)



          For simplicity, this particular confidence interval takes $sigma_n$ as known. If this is unknown then you would need to specify its relationship to the sample variance and you would probably then derive a similar interval, only using the critical values of the T-distribution instead of the normal distribution.






          share|cite|improve this answer











          $endgroup$













          • $begingroup$
            I'm aware of this, but I guess I should have mentioned that I'm trying to get a confidence interval that's independent of $theta$. Recall that if we have $X_1, ldots, X_n sim mathcal{N}(mu, sigma^2)$, we have that $mathbb{P}(|overline{X}_n - mu| > varepsilon) < 2e^{-nvarepsilon^2/2sigma^2}$, and thus we can invert the above to have a $geq 1 - delta$ confidence inverval that doesn't depend on $mu$. So what I'm trying to do here is somehow using the consistency information about $hat{theta}_n$ to get something independent of $theta$.
            $endgroup$
            – user610084
            Nov 30 '18 at 13:54












          • $begingroup$
            The same thing happens here - the above confidence interval is an interval for $theta$ but it doesn't use $theta$ in the formula (otherwise you couldn't calculate it from the observed data). As you can see from the final formula for the confidence interval, it only uses $hat{theta}_n$ (which is a statistic).
            $endgroup$
            – Ben
            Nov 30 '18 at 22:49










          • $begingroup$
            I see. That makes sense. I then have a followup question. Suppose the problem setting is exactly the same, but everything is now multivariate. Can we find confidence intervals in each dimension? Dude to the fact that now everything is multivariate, inverting $f_n$ will be infeasible since we now have projections at play.
            $endgroup$
            – user610084
            Dec 3 '18 at 0:38










          • $begingroup$
            If the extension is to the multivariate normal then you will still have univariate normal for the marginal distributions, so you could just apply the same method in a single dimension. If you want a confidence interval for the vector parameter then you would form something analogous to the ellipses you get from a multivariate distribution, projected back with the inverse transformation. Getting this would be non-trivial.
            $endgroup$
            – Ben
            Dec 3 '18 at 1:46











          Your Answer





          StackExchange.ifUsing("editor", function () {
          return StackExchange.using("mathjaxEditing", function () {
          StackExchange.MarkdownEditor.creationCallbacks.add(function (editor, postfix) {
          StackExchange.mathjaxEditing.prepareWmdForMathJax(editor, postfix, [["$", "$"], ["\\(","\\)"]]);
          });
          });
          }, "mathjax-editing");

          StackExchange.ready(function() {
          var channelOptions = {
          tags: "".split(" "),
          id: "69"
          };
          initTagRenderer("".split(" "), "".split(" "), channelOptions);

          StackExchange.using("externalEditor", function() {
          // Have to fire editor after snippets, if snippets enabled
          if (StackExchange.settings.snippets.snippetsEnabled) {
          StackExchange.using("snippets", function() {
          createEditor();
          });
          }
          else {
          createEditor();
          }
          });

          function createEditor() {
          StackExchange.prepareEditor({
          heartbeatType: 'answer',
          autoActivateHeartbeat: false,
          convertImagesToLinks: true,
          noModals: true,
          showLowRepImageUploadWarning: true,
          reputationToPostImages: 10,
          bindNavPrevention: true,
          postfix: "",
          imageUploader: {
          brandingHtml: "Powered by u003ca class="icon-imgur-white" href="https://imgur.com/"u003eu003c/au003e",
          contentPolicyHtml: "User contributions licensed under u003ca href="https://creativecommons.org/licenses/by-sa/3.0/"u003ecc by-sa 3.0 with attribution requiredu003c/au003e u003ca href="https://stackoverflow.com/legal/content-policy"u003e(content policy)u003c/au003e",
          allowUrls: true
          },
          noCode: true, onDemand: true,
          discardSelector: ".discard-answer"
          ,immediatelyShowMarkdownHelp:true
          });


          }
          });














          draft saved

          draft discarded


















          StackExchange.ready(
          function () {
          StackExchange.openid.initPostLogin('.new-post-login', 'https%3a%2f%2fmath.stackexchange.com%2fquestions%2f3019636%2fconfidence-interval-of-a-biased-estimator-thats-asymptotiacally-unbiased%23new-answer', 'question_page');
          }
          );

          Post as a guest















          Required, but never shown
























          1 Answer
          1






          active

          oldest

          votes








          1 Answer
          1






          active

          oldest

          votes









          active

          oldest

          votes






          active

          oldest

          votes









          0












          $begingroup$

          At the moment your stated distribution for $hat{theta}_n$ does not specify the relationship between the moments $mu_n$ and $sigma_n^2$ and the true parameter $theta$. This means that there is presently insufficient information to form a pivotal quantity that would be the basis for a confidence interval. If we make some assumptions about the relationship of these moments to the underlying parameter then we can form a pivotal quantity and derive an appropriate confidence interval formula.



          For simplicity, I will assume that the variance $sigma_n^2$ is unrelated to the underlying parameter, and that the mean is related to the underlying parameter through a monotonic function. That is, you have $mu_n = f_n(theta)$ via some monotonically increasing function $f_n$ (we will denote the inverse function by $g_n = f_n^{-1}$). Then you can write the pivotal quantity:



          $$frac{hat{theta}_n - f_n(theta)}{sigma_n} sim mathcal{N}(0,1).$$



          You have:



          $$begin{equation} begin{aligned}
          1-alpha
          &= mathbb{P} Big( z_{alpha/2} leqslant frac{hat{theta}_n - f_n(theta)}{sigma_n} leqslant z_{alpha/2} Big) \[6pt]
          &= mathbb{P} Big( sigma_n cdot z_{alpha/2} leqslant hat{theta}_n - f_n(theta) leqslant sigma_n cdot z_{alpha/2} Big) \[6pt]
          &= mathbb{P} Big( hat{theta}_n - sigma_n cdot z_{alpha/2} leqslant f_n(theta) leqslant hat{theta}_n + sigma_n cdot z_{alpha/2} Big) \[6pt]
          &= mathbb{P} Big( g_n(hat{theta}_n - sigma_n cdot z_{alpha/2}) leqslant theta leqslant g_n(hat{theta}_n + sigma_n cdot z_{alpha/2}) Big). \[6pt]
          end{aligned} end{equation}$$



          In the above equations we have taken $hat{theta}_n$ to be a random variable (i.e., prior to fixing the data). Once the data is observed this statistic becomes a fixed value $hat{theta}_{n,text{obs}}$ and we have the $1-alpha$ level confidence interval:



          $$text{CI}_theta(1-alpha) = Big[ g_n(hat{theta}_{n,text{obs}} - sigma_n cdot z_{alpha/2}), text{ } g_n(hat{theta}_{n,text{obs}} + sigma_n cdot z_{alpha/2}) Big].$$



          (Note that this formula is for a monotonically increasing function $f_n$. If the function were instead monotonically decreasing then the bounds of the confidence interval would be reversed.)



          For simplicity, this particular confidence interval takes $sigma_n$ as known. If this is unknown then you would need to specify its relationship to the sample variance and you would probably then derive a similar interval, only using the critical values of the T-distribution instead of the normal distribution.






          share|cite|improve this answer











          $endgroup$













          • $begingroup$
            I'm aware of this, but I guess I should have mentioned that I'm trying to get a confidence interval that's independent of $theta$. Recall that if we have $X_1, ldots, X_n sim mathcal{N}(mu, sigma^2)$, we have that $mathbb{P}(|overline{X}_n - mu| > varepsilon) < 2e^{-nvarepsilon^2/2sigma^2}$, and thus we can invert the above to have a $geq 1 - delta$ confidence inverval that doesn't depend on $mu$. So what I'm trying to do here is somehow using the consistency information about $hat{theta}_n$ to get something independent of $theta$.
            $endgroup$
            – user610084
            Nov 30 '18 at 13:54












          • $begingroup$
            The same thing happens here - the above confidence interval is an interval for $theta$ but it doesn't use $theta$ in the formula (otherwise you couldn't calculate it from the observed data). As you can see from the final formula for the confidence interval, it only uses $hat{theta}_n$ (which is a statistic).
            $endgroup$
            – Ben
            Nov 30 '18 at 22:49










          • $begingroup$
            I see. That makes sense. I then have a followup question. Suppose the problem setting is exactly the same, but everything is now multivariate. Can we find confidence intervals in each dimension? Dude to the fact that now everything is multivariate, inverting $f_n$ will be infeasible since we now have projections at play.
            $endgroup$
            – user610084
            Dec 3 '18 at 0:38










          • $begingroup$
            If the extension is to the multivariate normal then you will still have univariate normal for the marginal distributions, so you could just apply the same method in a single dimension. If you want a confidence interval for the vector parameter then you would form something analogous to the ellipses you get from a multivariate distribution, projected back with the inverse transformation. Getting this would be non-trivial.
            $endgroup$
            – Ben
            Dec 3 '18 at 1:46
















          0












          $begingroup$

          At the moment your stated distribution for $hat{theta}_n$ does not specify the relationship between the moments $mu_n$ and $sigma_n^2$ and the true parameter $theta$. This means that there is presently insufficient information to form a pivotal quantity that would be the basis for a confidence interval. If we make some assumptions about the relationship of these moments to the underlying parameter then we can form a pivotal quantity and derive an appropriate confidence interval formula.



          For simplicity, I will assume that the variance $sigma_n^2$ is unrelated to the underlying parameter, and that the mean is related to the underlying parameter through a monotonic function. That is, you have $mu_n = f_n(theta)$ via some monotonically increasing function $f_n$ (we will denote the inverse function by $g_n = f_n^{-1}$). Then you can write the pivotal quantity:



          $$frac{hat{theta}_n - f_n(theta)}{sigma_n} sim mathcal{N}(0,1).$$



          You have:



          $$begin{equation} begin{aligned}
          1-alpha
          &= mathbb{P} Big( z_{alpha/2} leqslant frac{hat{theta}_n - f_n(theta)}{sigma_n} leqslant z_{alpha/2} Big) \[6pt]
          &= mathbb{P} Big( sigma_n cdot z_{alpha/2} leqslant hat{theta}_n - f_n(theta) leqslant sigma_n cdot z_{alpha/2} Big) \[6pt]
          &= mathbb{P} Big( hat{theta}_n - sigma_n cdot z_{alpha/2} leqslant f_n(theta) leqslant hat{theta}_n + sigma_n cdot z_{alpha/2} Big) \[6pt]
          &= mathbb{P} Big( g_n(hat{theta}_n - sigma_n cdot z_{alpha/2}) leqslant theta leqslant g_n(hat{theta}_n + sigma_n cdot z_{alpha/2}) Big). \[6pt]
          end{aligned} end{equation}$$



          In the above equations we have taken $hat{theta}_n$ to be a random variable (i.e., prior to fixing the data). Once the data is observed this statistic becomes a fixed value $hat{theta}_{n,text{obs}}$ and we have the $1-alpha$ level confidence interval:



          $$text{CI}_theta(1-alpha) = Big[ g_n(hat{theta}_{n,text{obs}} - sigma_n cdot z_{alpha/2}), text{ } g_n(hat{theta}_{n,text{obs}} + sigma_n cdot z_{alpha/2}) Big].$$



          (Note that this formula is for a monotonically increasing function $f_n$. If the function were instead monotonically decreasing then the bounds of the confidence interval would be reversed.)



          For simplicity, this particular confidence interval takes $sigma_n$ as known. If this is unknown then you would need to specify its relationship to the sample variance and you would probably then derive a similar interval, only using the critical values of the T-distribution instead of the normal distribution.






          share|cite|improve this answer











          $endgroup$













          • $begingroup$
            I'm aware of this, but I guess I should have mentioned that I'm trying to get a confidence interval that's independent of $theta$. Recall that if we have $X_1, ldots, X_n sim mathcal{N}(mu, sigma^2)$, we have that $mathbb{P}(|overline{X}_n - mu| > varepsilon) < 2e^{-nvarepsilon^2/2sigma^2}$, and thus we can invert the above to have a $geq 1 - delta$ confidence inverval that doesn't depend on $mu$. So what I'm trying to do here is somehow using the consistency information about $hat{theta}_n$ to get something independent of $theta$.
            $endgroup$
            – user610084
            Nov 30 '18 at 13:54












          • $begingroup$
            The same thing happens here - the above confidence interval is an interval for $theta$ but it doesn't use $theta$ in the formula (otherwise you couldn't calculate it from the observed data). As you can see from the final formula for the confidence interval, it only uses $hat{theta}_n$ (which is a statistic).
            $endgroup$
            – Ben
            Nov 30 '18 at 22:49










          • $begingroup$
            I see. That makes sense. I then have a followup question. Suppose the problem setting is exactly the same, but everything is now multivariate. Can we find confidence intervals in each dimension? Dude to the fact that now everything is multivariate, inverting $f_n$ will be infeasible since we now have projections at play.
            $endgroup$
            – user610084
            Dec 3 '18 at 0:38










          • $begingroup$
            If the extension is to the multivariate normal then you will still have univariate normal for the marginal distributions, so you could just apply the same method in a single dimension. If you want a confidence interval for the vector parameter then you would form something analogous to the ellipses you get from a multivariate distribution, projected back with the inverse transformation. Getting this would be non-trivial.
            $endgroup$
            – Ben
            Dec 3 '18 at 1:46














          0












          0








          0





          $begingroup$

          At the moment your stated distribution for $hat{theta}_n$ does not specify the relationship between the moments $mu_n$ and $sigma_n^2$ and the true parameter $theta$. This means that there is presently insufficient information to form a pivotal quantity that would be the basis for a confidence interval. If we make some assumptions about the relationship of these moments to the underlying parameter then we can form a pivotal quantity and derive an appropriate confidence interval formula.



          For simplicity, I will assume that the variance $sigma_n^2$ is unrelated to the underlying parameter, and that the mean is related to the underlying parameter through a monotonic function. That is, you have $mu_n = f_n(theta)$ via some monotonically increasing function $f_n$ (we will denote the inverse function by $g_n = f_n^{-1}$). Then you can write the pivotal quantity:



          $$frac{hat{theta}_n - f_n(theta)}{sigma_n} sim mathcal{N}(0,1).$$



          You have:



          $$begin{equation} begin{aligned}
          1-alpha
          &= mathbb{P} Big( z_{alpha/2} leqslant frac{hat{theta}_n - f_n(theta)}{sigma_n} leqslant z_{alpha/2} Big) \[6pt]
          &= mathbb{P} Big( sigma_n cdot z_{alpha/2} leqslant hat{theta}_n - f_n(theta) leqslant sigma_n cdot z_{alpha/2} Big) \[6pt]
          &= mathbb{P} Big( hat{theta}_n - sigma_n cdot z_{alpha/2} leqslant f_n(theta) leqslant hat{theta}_n + sigma_n cdot z_{alpha/2} Big) \[6pt]
          &= mathbb{P} Big( g_n(hat{theta}_n - sigma_n cdot z_{alpha/2}) leqslant theta leqslant g_n(hat{theta}_n + sigma_n cdot z_{alpha/2}) Big). \[6pt]
          end{aligned} end{equation}$$



          In the above equations we have taken $hat{theta}_n$ to be a random variable (i.e., prior to fixing the data). Once the data is observed this statistic becomes a fixed value $hat{theta}_{n,text{obs}}$ and we have the $1-alpha$ level confidence interval:



          $$text{CI}_theta(1-alpha) = Big[ g_n(hat{theta}_{n,text{obs}} - sigma_n cdot z_{alpha/2}), text{ } g_n(hat{theta}_{n,text{obs}} + sigma_n cdot z_{alpha/2}) Big].$$



          (Note that this formula is for a monotonically increasing function $f_n$. If the function were instead monotonically decreasing then the bounds of the confidence interval would be reversed.)



          For simplicity, this particular confidence interval takes $sigma_n$ as known. If this is unknown then you would need to specify its relationship to the sample variance and you would probably then derive a similar interval, only using the critical values of the T-distribution instead of the normal distribution.






          share|cite|improve this answer











          $endgroup$



          At the moment your stated distribution for $hat{theta}_n$ does not specify the relationship between the moments $mu_n$ and $sigma_n^2$ and the true parameter $theta$. This means that there is presently insufficient information to form a pivotal quantity that would be the basis for a confidence interval. If we make some assumptions about the relationship of these moments to the underlying parameter then we can form a pivotal quantity and derive an appropriate confidence interval formula.



          For simplicity, I will assume that the variance $sigma_n^2$ is unrelated to the underlying parameter, and that the mean is related to the underlying parameter through a monotonic function. That is, you have $mu_n = f_n(theta)$ via some monotonically increasing function $f_n$ (we will denote the inverse function by $g_n = f_n^{-1}$). Then you can write the pivotal quantity:



          $$frac{hat{theta}_n - f_n(theta)}{sigma_n} sim mathcal{N}(0,1).$$



          You have:



          $$begin{equation} begin{aligned}
          1-alpha
          &= mathbb{P} Big( z_{alpha/2} leqslant frac{hat{theta}_n - f_n(theta)}{sigma_n} leqslant z_{alpha/2} Big) \[6pt]
          &= mathbb{P} Big( sigma_n cdot z_{alpha/2} leqslant hat{theta}_n - f_n(theta) leqslant sigma_n cdot z_{alpha/2} Big) \[6pt]
          &= mathbb{P} Big( hat{theta}_n - sigma_n cdot z_{alpha/2} leqslant f_n(theta) leqslant hat{theta}_n + sigma_n cdot z_{alpha/2} Big) \[6pt]
          &= mathbb{P} Big( g_n(hat{theta}_n - sigma_n cdot z_{alpha/2}) leqslant theta leqslant g_n(hat{theta}_n + sigma_n cdot z_{alpha/2}) Big). \[6pt]
          end{aligned} end{equation}$$



          In the above equations we have taken $hat{theta}_n$ to be a random variable (i.e., prior to fixing the data). Once the data is observed this statistic becomes a fixed value $hat{theta}_{n,text{obs}}$ and we have the $1-alpha$ level confidence interval:



          $$text{CI}_theta(1-alpha) = Big[ g_n(hat{theta}_{n,text{obs}} - sigma_n cdot z_{alpha/2}), text{ } g_n(hat{theta}_{n,text{obs}} + sigma_n cdot z_{alpha/2}) Big].$$



          (Note that this formula is for a monotonically increasing function $f_n$. If the function were instead monotonically decreasing then the bounds of the confidence interval would be reversed.)



          For simplicity, this particular confidence interval takes $sigma_n$ as known. If this is unknown then you would need to specify its relationship to the sample variance and you would probably then derive a similar interval, only using the critical values of the T-distribution instead of the normal distribution.







          share|cite|improve this answer














          share|cite|improve this answer



          share|cite|improve this answer








          edited Nov 30 '18 at 22:51

























          answered Nov 30 '18 at 5:33









          BenBen

          1,118115




          1,118115












          • $begingroup$
            I'm aware of this, but I guess I should have mentioned that I'm trying to get a confidence interval that's independent of $theta$. Recall that if we have $X_1, ldots, X_n sim mathcal{N}(mu, sigma^2)$, we have that $mathbb{P}(|overline{X}_n - mu| > varepsilon) < 2e^{-nvarepsilon^2/2sigma^2}$, and thus we can invert the above to have a $geq 1 - delta$ confidence inverval that doesn't depend on $mu$. So what I'm trying to do here is somehow using the consistency information about $hat{theta}_n$ to get something independent of $theta$.
            $endgroup$
            – user610084
            Nov 30 '18 at 13:54












          • $begingroup$
            The same thing happens here - the above confidence interval is an interval for $theta$ but it doesn't use $theta$ in the formula (otherwise you couldn't calculate it from the observed data). As you can see from the final formula for the confidence interval, it only uses $hat{theta}_n$ (which is a statistic).
            $endgroup$
            – Ben
            Nov 30 '18 at 22:49










          • $begingroup$
            I see. That makes sense. I then have a followup question. Suppose the problem setting is exactly the same, but everything is now multivariate. Can we find confidence intervals in each dimension? Dude to the fact that now everything is multivariate, inverting $f_n$ will be infeasible since we now have projections at play.
            $endgroup$
            – user610084
            Dec 3 '18 at 0:38










          • $begingroup$
            If the extension is to the multivariate normal then you will still have univariate normal for the marginal distributions, so you could just apply the same method in a single dimension. If you want a confidence interval for the vector parameter then you would form something analogous to the ellipses you get from a multivariate distribution, projected back with the inverse transformation. Getting this would be non-trivial.
            $endgroup$
            – Ben
            Dec 3 '18 at 1:46


















          • $begingroup$
            I'm aware of this, but I guess I should have mentioned that I'm trying to get a confidence interval that's independent of $theta$. Recall that if we have $X_1, ldots, X_n sim mathcal{N}(mu, sigma^2)$, we have that $mathbb{P}(|overline{X}_n - mu| > varepsilon) < 2e^{-nvarepsilon^2/2sigma^2}$, and thus we can invert the above to have a $geq 1 - delta$ confidence inverval that doesn't depend on $mu$. So what I'm trying to do here is somehow using the consistency information about $hat{theta}_n$ to get something independent of $theta$.
            $endgroup$
            – user610084
            Nov 30 '18 at 13:54












          • $begingroup$
            The same thing happens here - the above confidence interval is an interval for $theta$ but it doesn't use $theta$ in the formula (otherwise you couldn't calculate it from the observed data). As you can see from the final formula for the confidence interval, it only uses $hat{theta}_n$ (which is a statistic).
            $endgroup$
            – Ben
            Nov 30 '18 at 22:49










          • $begingroup$
            I see. That makes sense. I then have a followup question. Suppose the problem setting is exactly the same, but everything is now multivariate. Can we find confidence intervals in each dimension? Dude to the fact that now everything is multivariate, inverting $f_n$ will be infeasible since we now have projections at play.
            $endgroup$
            – user610084
            Dec 3 '18 at 0:38










          • $begingroup$
            If the extension is to the multivariate normal then you will still have univariate normal for the marginal distributions, so you could just apply the same method in a single dimension. If you want a confidence interval for the vector parameter then you would form something analogous to the ellipses you get from a multivariate distribution, projected back with the inverse transformation. Getting this would be non-trivial.
            $endgroup$
            – Ben
            Dec 3 '18 at 1:46
















          $begingroup$
          I'm aware of this, but I guess I should have mentioned that I'm trying to get a confidence interval that's independent of $theta$. Recall that if we have $X_1, ldots, X_n sim mathcal{N}(mu, sigma^2)$, we have that $mathbb{P}(|overline{X}_n - mu| > varepsilon) < 2e^{-nvarepsilon^2/2sigma^2}$, and thus we can invert the above to have a $geq 1 - delta$ confidence inverval that doesn't depend on $mu$. So what I'm trying to do here is somehow using the consistency information about $hat{theta}_n$ to get something independent of $theta$.
          $endgroup$
          – user610084
          Nov 30 '18 at 13:54






          $begingroup$
          I'm aware of this, but I guess I should have mentioned that I'm trying to get a confidence interval that's independent of $theta$. Recall that if we have $X_1, ldots, X_n sim mathcal{N}(mu, sigma^2)$, we have that $mathbb{P}(|overline{X}_n - mu| > varepsilon) < 2e^{-nvarepsilon^2/2sigma^2}$, and thus we can invert the above to have a $geq 1 - delta$ confidence inverval that doesn't depend on $mu$. So what I'm trying to do here is somehow using the consistency information about $hat{theta}_n$ to get something independent of $theta$.
          $endgroup$
          – user610084
          Nov 30 '18 at 13:54














          $begingroup$
          The same thing happens here - the above confidence interval is an interval for $theta$ but it doesn't use $theta$ in the formula (otherwise you couldn't calculate it from the observed data). As you can see from the final formula for the confidence interval, it only uses $hat{theta}_n$ (which is a statistic).
          $endgroup$
          – Ben
          Nov 30 '18 at 22:49




          $begingroup$
          The same thing happens here - the above confidence interval is an interval for $theta$ but it doesn't use $theta$ in the formula (otherwise you couldn't calculate it from the observed data). As you can see from the final formula for the confidence interval, it only uses $hat{theta}_n$ (which is a statistic).
          $endgroup$
          – Ben
          Nov 30 '18 at 22:49












          $begingroup$
          I see. That makes sense. I then have a followup question. Suppose the problem setting is exactly the same, but everything is now multivariate. Can we find confidence intervals in each dimension? Dude to the fact that now everything is multivariate, inverting $f_n$ will be infeasible since we now have projections at play.
          $endgroup$
          – user610084
          Dec 3 '18 at 0:38




          $begingroup$
          I see. That makes sense. I then have a followup question. Suppose the problem setting is exactly the same, but everything is now multivariate. Can we find confidence intervals in each dimension? Dude to the fact that now everything is multivariate, inverting $f_n$ will be infeasible since we now have projections at play.
          $endgroup$
          – user610084
          Dec 3 '18 at 0:38












          $begingroup$
          If the extension is to the multivariate normal then you will still have univariate normal for the marginal distributions, so you could just apply the same method in a single dimension. If you want a confidence interval for the vector parameter then you would form something analogous to the ellipses you get from a multivariate distribution, projected back with the inverse transformation. Getting this would be non-trivial.
          $endgroup$
          – Ben
          Dec 3 '18 at 1:46




          $begingroup$
          If the extension is to the multivariate normal then you will still have univariate normal for the marginal distributions, so you could just apply the same method in a single dimension. If you want a confidence interval for the vector parameter then you would form something analogous to the ellipses you get from a multivariate distribution, projected back with the inverse transformation. Getting this would be non-trivial.
          $endgroup$
          – Ben
          Dec 3 '18 at 1:46


















          draft saved

          draft discarded




















































          Thanks for contributing an answer to Mathematics Stack Exchange!


          • Please be sure to answer the question. Provide details and share your research!

          But avoid



          • Asking for help, clarification, or responding to other answers.

          • Making statements based on opinion; back them up with references or personal experience.


          Use MathJax to format equations. MathJax reference.


          To learn more, see our tips on writing great answers.




          draft saved


          draft discarded














          StackExchange.ready(
          function () {
          StackExchange.openid.initPostLogin('.new-post-login', 'https%3a%2f%2fmath.stackexchange.com%2fquestions%2f3019636%2fconfidence-interval-of-a-biased-estimator-thats-asymptotiacally-unbiased%23new-answer', 'question_page');
          }
          );

          Post as a guest















          Required, but never shown





















































          Required, but never shown














          Required, but never shown












          Required, but never shown







          Required, but never shown

































          Required, but never shown














          Required, but never shown












          Required, but never shown







          Required, but never shown







          Popular posts from this blog

          Biblatex bibliography style without URLs when DOI exists (in Overleaf with Zotero bibliography)

          ComboBox Display Member on multiple fields

          Is it possible to collect Nectar points via Trainline?